cancel
Showing results for 
Show  only  | Search instead for 
Did you mean: 
JMP Wish List

We want to hear your ideas for improving JMP software.

  1. Search: Please search for an existing idea first before submitting a new idea.
  2. Submit: Post your new idea using the Suggest an Idea button. Please submit one actionable idea per post rather than a single post with multiple ideas.
  3. Kudo & Comment Kudo ideas you like, and comment to add to an idea.
  4. Subscribe: Follow the status of ideas you like. Refer to status definitions to understand where an idea is in its lifecycle. (You are automatically subscribed to ideas you've submitted or commented on.)

We consider several factors when looking for what ideas to add to JMP. This includes what will have the greatest benefit to our customers based on scope, needs and current resources. Product ideas help us decide what features to work on next. Additionally, we often look to ideas for inspiration on how to add value to developments already in our pipeline or enhancements to new or existing features.

Choose Language Hide Translation Bar

Paired t-test in Fit Y by X Platform with Unbalanced Data

We teach many of our engineers about paired t-tests and use the Fit Y by X platform (rather than Matched Pairs) when the format of their data set is more suitable for Fit Y by X. There are 2 frustrating aspects:

 

  1. There is still an option for 't test' in the analysis, which does an ordinary two-sample t-test, which ignores the Block that you incorporated. Most engineers/scientists don't realize this.
  2. If the data is unbalanced by even 1 data point (e.g. I have 20 pairs, but there is 1 missing data point or outlier that was discarded), you get an error message saying the Blocking is going to be ignored. It's a somewhat confusing message for them, and I feel that most engineers/scientists just blindly keep following the platform output. If I had fit the same model using Fit Model, this slight unbalance in the data can easily be utilized to give the correct hypothesis testing output. I'd like to see the Fit Y by X output handle the unbalanced data when the correct model is rather straightforward.
2 Comments
Jeff_Perkinson
Community Manager
Status changed to: Yes, Stay Tuned!

We are working on this for JMP 16.

Jeff_Perkinson
Community Manager
Status changed to: Delivered

In JMP 16 when a Block variable is specified and the cell counts are unequal, the Oneway report contains Fit Model Standard Least Squares output to accommodate the blocking.